Quiz Summary
0 of 50 Questions completed
Questions:
Information
You have already completed the quiz before. Hence you can not start it again.
Quiz is loading…
You must sign in or sign up to start the quiz.
You must first complete the following:
Results
Results
0 of 50 Questions answered correctly
Your time:
Time has elapsed
You have reached 0 of 0 point(s), (0)
Earned Point(s): 0 of 0, (0)
0 Essay(s) Pending (Possible Point(s): 0)
Average score |
|
Your score |
|
Categories
- NBME 18 BLOCK 4 0%
Pos. | Name | Entered on | Points | Result |
---|---|---|---|---|
Table is loading | ||||
No data available | ||||
- 1
- 2
- 3
- 4
- 5
- 6
- 7
- 8
- 9
- 10
- 11
- 12
- 13
- 14
- 15
- 16
- 17
- 18
- 19
- 20
- 21
- 22
- 23
- 24
- 25
- 26
- 27
- 28
- 29
- 30
- 31
- 32
- 33
- 34
- 35
- 36
- 37
- 38
- 39
- 40
- 41
- 42
- 43
- 44
- 45
- 46
- 47
- 48
- 49
- 50
- Current
- Review
- Answered
- Correct
- Incorrect
-
Question 1 of 50
1. Question
A 6-week-old male infant has a persistent discharge from a swollen umbilical stump. If the discharge contains intestinal fluid, to which of the following portions of the gastrointestinal tract is the fistula most likely connected?
CorrectIncorrect -
Question 2 of 50
2. Question
A 5-year-old boy is brought to the physician by his parents because of lethargy for 1 week. He has a history of occasional colds and ear infections. Physical examination shows mild pitting edema of the lower extremities. Auscultation of the chest and palpation of the abdomen show no abnormalities. Laboratory studies show:
If a kidney biopsy specimen were obtained, light microscopic examination would most likely show which of the following?
CorrectIncorrect -
Question 3 of 50
3. Question
Strips of tracheal muscle contract after administration of carbachol then relax after administration of epinephrine. The graph shows muscle tension as a function of the log dose of epinephrine under control conditions (solid curve) and in the presence of equal concentrations of antagonists X (dashed curve) and Y (dotted curve). X and Y are most likely to be which of the following drugs?
CorrectIncorrect -
Question 4 of 50
4. Question
A 58-year-old woman is brought to the emergency department 6 hours after the sudden onset of left-sided facial, arm, and leg weakness. She has hypertension treated with a diuretic. She reports the sensation of a fluttering heart for 1 week. Her pulse is 125/min and irregular, and blood pressure is 135/80 mm Hg. Physical examination shows a left¬sided facial droop. There is flaccid weakness of the left upper and lower extremities. An ECG shows a rapid, irregular ventricular rate with no discernable P waves. Which of the following therapies would best improve function of the weakened muscles over the next 3 hours?
CorrectIncorrect -
Question 5 of 50
5. Question
When control lymphocytes are treated with corticosteroids, a majority of cells shrink in size and develop peripheral chromatin condensation; cytoplasmic organelles are intact. DNA isolated from the control lymphocytes, electrophoresed on an agarose gel and stained with ethidium bromide, shows a ladder of regularly spaced bands. When lymphocytes from the same culture are transfected so that they overexpress Gene X, which encodes a normal protein product, the cells continue to proliferate and do not undergo any of the morphologic changes seen in the control cells. Gene X is most likely to encode which of the following?
CorrectIncorrect -
Question 6 of 50
6. Question
A 1-year-old African American girl is brought to the physician because of a slow growth rate. At birth she was at the 25th percentile for length and 30th percentile for weight. She has been exclusively breast-fed since birth. Her length and weight are now at the 5th percentile. Physical examination shows bowed legs, swelling of the wrists, and a bulging of the costochondral junctions. Her serum calcium concentration is within the reference range, and her serum phosphorus concentration is decreased. Which of the following sets of serum findings is most likely in this patient?
CorrectIncorrect -
Question 7 of 50
7. Question
A 70-year-old woman is brought to the emergency department 30 minutes after she was found unresponsive at home. She appears stuporous. Physical examination shows 2-mm-diameter pupils that are reactive to light. Eye movements are full with ice water caloric stimulation. She withdraws the extremities symmetrically to painful stimuli. An MRI of the brain is shown; the arrowheads indicate abnormalities. Which of the following is most likely to be present in this patient 2 months later?
CorrectIncorrect -
Question 8 of 50
8. Question
A 25-year-old woman comes to the physician because of general malaise and a facial rash for 1 week. She has a 10-year history of episodes of pleurisy and arthritic pain in peripheral joints. Physical examination shows an erythematous malar rash that does not involve the nasolabial folds. Results of cardiolipin antibody, anti-dsDNA, and anti-Sm antibody assays are positive. Which of the following hematologic abnormalities is most likely in this patient?
CorrectIncorrect -
Question 9 of 50
9. Question
An 18-month-old boy is brought to the physician by his parents because of eye pain for 3 days. He has a history of multiple infections since the age of 3 weeks, including upper respiratory tract infections, staphylococcal osteomyelitis, and two episodes of pneumonia (one due to Staphylococcus aureus and the other due to Aspergillus fumigatus). Ophthalmologic examination shows erythema of the conjunctiva of the left eye with a small amount of purulent discharge. Physical examination shows multiple ulcerated lesions with erythematous borders over the face, neck, and back; cervical and axillary lymphadenopathy; and hepatosplenomegaly. Serum IgE, IgG, and IgM concentrations are within the reference ranges. Culture of the conjunctival secretions grows Staphylococcus epidermidis, and culture of several skin lesions grows Staphylococcus aureus. This patient most likely has which of the following immunodeficiency diseases?
CorrectIncorrect -
Question 10 of 50
10. Question
An investigator is planning to create gene therapy for Leigh syndrome, which is caused by an A—>G mutation in the mitochondrial tRNALeu gene. Which of the following is the most likely reason why mitochondria encode their own tRNA?
CorrectIncorrect -
Question 11 of 50
11. Question
A 55-year-old man comes to the physician because of a 3-week history of night sweats, weakness, fatigue, loss of appetite, and lumps in his neck. He owns a printing business and works with paints and solvents. Physical examination shows edematous, tender axillary and inguinal lymph nodes. His leukocyte count is 45,570/mm3 (80% large, pleomorphic blasts with Auer rods). Occupational exposure to which of the following most likely contributed to the development of this patient’s disease?
CorrectIncorrect -
Question 12 of 50
12. Question
Feedback inhibition of the activity of the enzyme that catalyzes this reaction is critical in the control of the de novo synthesis of which of the following?
CorrectIncorrect -
Question 13 of 50
13. Question
An investigator is conducting a study of the lipopolysaccharide synthesis pathway. During the study, a polymerase chain
reaction test is developed that detects a gene involved in
lipid A biosynthesis. Which of the following organisms is most likely to yield a positive result using this polymerase chain reaction test?
CorrectIncorrect -
Question 14 of 50
14. Question
As part of an annual physical examination, a healthy 50-year-old man has laboratory studies done 8 hours after an overnight fast. His serum glucose concentration is 75 mg/dL. Activation of which of the following receptors in liver cells most likely maintained the serum glucose concentration in this patient?
CorrectIncorrect -
Question 15 of 50
15. Question
A 65-year-old man comes to the physician for a follow-up examination. He has a 15-year history of poorly controlled type 2 diabetes mellitus, resulting in multiple peripheral neuropathies. Neurologic examination shows wasting of the interosseous muscles of the left hand and inability to abduct the fingers of this hand; plantar flexion of the right foot is absent. The function of which of the following pairs of nerves is most likely impaired in this patient?
CorrectIncorrect -
Question 16 of 50
16. Question
A 52-year-old woman is admitted to the hospital because of chest pain and shortness of breath for 2 hours. Her pulse is 102/min, respirations are 35/min, and blood pressure is 110/65 mm Hg. Cardiac examination shows a laterally displaced apex, a systolic thrill, and a grade 4/6, crescendo-decrescendo systolic ejection murmur. Cardiac catheterization shows a pulmonary capillary wedge pressure of 40 mm Hg (N=5-16). Which of the following sets of changes in this patient’s left ventricle is most likely?
CorrectIncorrect -
Question 17 of 50
17. Question
A 22-year-old man comes to the physician because of a 2-month history of foul-smelling, watery diarrhea with significant flatulence; he also has had a 4.5-kg (10-lb) weight loss during this period. He recently returned from a trip to rural Indonesia, where he did not always have access to clean water. He appears thin. Physical examination shows a soft, mildly distended abdomen with active bowel sounds. A photomicrograph of a stool specimen is shown. The most appropriate pharmacotherapy has which of the following mechanisms of action?
CorrectIncorrect -
Question 18 of 50
18. Question
A 68-year-old man with type 2 diabetes mellitus, hypertension, and hyperlipidemia comes to the physician because of pain with urination, blood in his urine, and fatigue during the past week. He has a 6-month history of increased urinary frequency with small volume and nighttime urination. He has had two episodes of acute cystitis during the past 8 months. His temperature is 38.3°C (100.9°F), pulse is 92/min, respirations are 20/min, and blood pressure is 108/72 mm Hg. Physical examination shows suprapubic tenderness and an enlarged, soft prostate. Ultrasonography shows a bladder diverticulum and enlarged kidneys with dilation of renal pelves and calyces. A urine culture grows Escherichia coli. Which of the following is the most likely underlying cause of this patient’s condition?
CorrectIncorrect -
Question 19 of 50
19. Question
A68-year-old man with type 2 diabetes mellitus, hypertension, and hyperlipidemia comes to the physician because of pain with urination, blood in his urine, and fatigue during the past week. He has a 6-month history of increased urinary frequency with small volume and nighttime urination. He has had two episodes of acute cystitis during the past 8 months. His temperature is 38.3°C (100.9°F), pulse is 92/min, respirations are 20/min, and blood pressure is 108/72 mm Hg. Physical examination shows suprapubic tenderness and an enlarged, soft prostate. Ultrasonography shows a bladder diverticulum and enlarged kidneys with dilation of renal pelves and calyces. A urine culture grows Escherichia coli. Which of the following is the most likely underlying cause of this patient’s condition?
CorrectIncorrect -
Question 20 of 50
20. Question
A 28-year-old woman comes to the physician because of a 1-week history of fatigue, muscle pain, and generalized weakness. She is unable to climb stairs because of the symptoms. She also has a 1-month history of a rash on her face, elbows, and knuckles. Her temperature is 38.3°C (100.9°F), pulse is 100/min, respirations are 20/min, and blood pressure is 115/65 mm Hg. Physical examination shows the findings in the photographs. There is also proximal muscle weakness of the lower extremities bilaterally. Which of the following is the most likely diagnosis?
CorrectIncorrect -
Question 21 of 50
21. Question
A male newborn is delivered vaginally at 39 weeks’ gestation to a 24-year-old primigravid woman. The pregnancy was complicated by polyhydramnios. Soon after birth, the newborn has frothing at the mouth, cyanosis, and respiratory distress. An unsuccessful attempt is made to pass a nasogastric tube. An x-ray shows an air-distended stomach. Which of the following is the most likely cause of these findings in this patient?
CorrectIncorrect -
Question 22 of 50
22. Question
A matched case-control study is conducted to assess the association between the frequent use of tanning beds during teenage years and the development of melanoma. A group of women with melanoma are matched with a group of cancer-free women of the same age and ethnicity. An odds ratio (OR) of 1.4 is obtained. Which of the following 95% confidence intervals for the OR is the most precise statistically significant interval?
CorrectIncorrect -
Question 23 of 50
23. Question
A 48-year-old man begins furosemide therapy for pedal edema associated with biventricular failure and hypertension. Five days later, the edema is not fully resolved, and his serum potassium concentration has decreased from 4.2 mEq/L to 3 mEq/L. A drug with which of the following actions should be added to this patient’s medication regimen?
CorrectIncorrect -
Question 24 of 50
24. Question
A 34-year-old woman comes to the physician because she has been feeling weepy and overwhelmed since delivering a healthy male newborn 6 weeks ago. She has had fatigue and irritability during this period, and she has had no interest in engaging in activities she used to enjoy prior to the birth of the baby. She also has had difficulty sleeping because she is always listening for sounds indicating that her baby is awake. She says, “My husband doesn’t help me at all with the baby. We argue all the time now.” She then says, “I feel guilty because I’m not enjoying my baby more. I don’t know if I will be a good mother or not.” Physical examination shows no abnormalities. Which of the following statements by the physician is most appropriate?
CorrectIncorrect -
Question 25 of 50
25. Question
A 19-year-old man comes to the emergency department because of increasingly severe shoulder and abdominal pain for 3 days. His temperature is 39°C (102.2°F). Physical examination shows signs of acute peritonitis. An abdominal x-ray is shown. Which of the following is the most likely cause of this patient’s current condition?
CorrectIncorrect -
Question 26 of 50
26. Question
A 32-year-old man comes to the physician because of a 6-month history of progressive drooping of the eyelids and facial weakness. Muscle weakness is exacerbated by use and relieved by rest. If a neoplasm is the cause of these findings, which of the following is the most likely location in this patient?
CorrectIncorrect -
Question 27 of 50
27. Question
A 65-year-old woman with severe sensorineural hearing loss undergoes surgical placement of a cochlear implant. This neural prosthesis converts sound energy to electrical signals, which results in stimulation of which of the following structures?
CorrectIncorrect -
Question 28 of 50
28. Question
A 27-year-old man comes to the physician smears, and tuberculate macroconidia are because of fever and cough for 1 month. An x-ray of the chest shows diffuse infiltrates. Tiny intracellular yeast forms are seen on direct seen after culture at room temperature. Which of the following exposures is most consistent with this patient’s illness?
CorrectIncorrect -
Question 29 of 50
29. Question
A single, oral, 6-mg dose of a short-acting (β2-agonist is studied to determine its pharmacokinetics and pharmacodynamics in young subjects with stable mild asthma. In this group, the Cmax/AUC following oral dosing varies by less than 8%. However, in 30% of subjects, the maximal FEV1 increase is 25% lower than that of the other subjects. Which of the following mechanisms best explains these observations in this subgroup?
CorrectIncorrect -
Question 30 of 50
30. Question
A 10-year-old boy is brought to the physician by his parents because of increased urinary frequency and progressive fatigue during the past 3 weeks. Physical examination shows no abnormalities. His fasting serum glucose concentration is 350 mg/dL. Urinalysis shows 4+ glucose. Which of the following mechanisms in the proximal tubule is the most likely cause of the glycosuria in this patient?
CorrectIncorrect -
Question 31 of 50
31. Question
A 13-year-old girl is brought to the physician because of a mass in the thoracic region that has been enlarging during the past month. She was placed in a foster home 1 year ago. She is at the 70th percentile for height and weight. Breast and pubic hair development is Tanner stage 2. Physical examination shows a 3-mm, mildly pigmented mass located in the midclavicular line at the 10th rib. The mass has a smooth surface, uniform outer border, and central papule. There are no other pigmented lesions. Which of the following is the most likely diagnosis?
CorrectIncorrect -
Question 32 of 50
32. Question
A 19-year-old woman comes to the physician 6 hours after the sudden onset of left-sided, cramping abdominal pain. She does not have nausea. She also has a 3-week history of pain with sexual intercourse, and she is sexually active with three male partners. Her last menses started 5 days ago. Current medication is an oral contraceptive. Her temperature is 38°C (100.4°F). Physical examination shows a normal cervix with no discharge. Bimanual examination shows tenderness and fullness on the left side compared with the right, and cervical motion tenderness. A urine pregnancy test result is negative. Which of the following structures is most likely affected by this patient’s current condition?
CorrectIncorrect -
Question 33 of 50
33. Question
A 5-year-old boy has severe cholestatic liver disease and poor bile secretion. This patient is also likely to have a deficiency of vitamin E because its absorption requires which of the following?
CorrectIncorrect -
Question 34 of 50
34. Question
A 35-year-old woman, gravida 1, para 1, comes to the physician for a follow-up examination 1 month after delivering a healthy male newborn. During her pregnancy, she was diagnosed with gestational diabetes. Her mother and sister also developed this condition during pregnancy. Today, laboratory studies show a serum glucose concentration of 100 mg/dl_. This patient most likely has decreased activity of which of the following enzymes?
CorrectIncorrect -
Question 35 of 50
35. Question
Which of the following characteristics of the glucocorticoid receptor is the most likely cause of the persistence of the pharmacologic effects of dexamethasone after the drug is eliminated from the body?
CorrectIncorrect -
Question 36 of 50
36. Question
A premenopausal 49-year-old woman asks her physician about her risk for osteoporosis. Her mother had osteoporosis and disabling bone fractures after menopause. The patient works as a gardener for a landscape service and plays tennis 3 times a week. She does not smoke, drinks 1 glass of wine a week, and takes no medications. Her weight is 55 kg (121 lb). Which of the following additional information is needed to evaluate her risk for osteoporosis?
CorrectIncorrect -
Question 37 of 50
37. Question
A 32-year-old woman is found to have panic disorder with agoraphobia. A drug is prescribed that activates benzodiazepine binding sites on the y-aminobutyric acidA (GABAa) receptor. This drug is most likely which of the following?
CorrectIncorrect -
Question 38 of 50
38. Question
A 60-year-old woman comes to the physician because of a 1-month history of bleeding from a lesion on her nose. She has no history of major medical illness and takes no medications. Physical examination shows a 1-cm lesion on the right naris. Microscopic examination of a biopsy specimen of the mass shows neoplastic cells that exhibit dense pigment granules. Which of the following is the most likely diagnosis?
CorrectIncorrect -
Question 39 of 50
39. Question
A 47-year-old man has microscopic blood on urinalysis. A CT scan of the abdomen shows a 5-cm mass in the left kidney. A photomicrograph of tissue obtained on renal biopsy is shown. Which of the following is the most likely diagnosis?
CorrectIncorrect -
Question 40 of 50
40. Question
A 28-year-old woman comes to the physician for advice on how to lose weight. She tells the physician that she binges on high-carbohydrate foods two to three times weekly, usually forcing herself to vomit after a binge. She is 168 cm (5 ft 6 in) tall and weighs 63 kg (140 lb); BMI is 23 kg/m2. Which of the following additional physical findings is most likely in this patient?
CorrectIncorrect -
Question 41 of 50
41. Question
A 22-year-old man is brought to the emergency department 30 minutes after a friend found him unconscious on the floor. A drug overdose is suspected. His pulse is 120/min, respirations are 4/min with apparent low tidal volumes, and blood pressure is 120/100 mm Hg. Physical examination shows mild cyanosis. He is intubated and mechanically ventilated with positive pressure ventilation (rate = 12/min; VT= 7 mL/kg) using 10 cm H20 positive end-expiratory pressure (PEEP). Which of the following sets of findings best describes the effects of the mechanical ventilation with PEEP on this patient’s alveolar (PA) and intrapleural (Pip) pressures?
CorrectIncorrect -
Question 42 of 50
42. Question
A 21 -year-old woman with primary pulmonary hypertension begins treatment with bosentan. As a result, blockade of which of the following is most likely to occur?
CorrectIncorrect -
Question 43 of 50
43. Question
A 12-year-old girl is brought to the physician because of a rash on her left buttock for the past 2 days. The rash developed after the family returned from a 2-week-long early summer vacation in Maine. Her vital signs are within normal limits. A photograph of the lesion is shown. The likely cause of this patient’s infection is taxonomically and morphologically most similar to the infectious agent of which of the following conditions?
CorrectIncorrect -
Question 44 of 50
44. Question
A 73-year-old man with stage IV colon cancer begins chemotherapy with 5-fluorouracil, irinotecan, and leucovorin. This patient is at greatest risk for developing signs of drug toxicity in which of the following tissues?
CorrectIncorrect -
Question 45 of 50
45. Question
A 5-month-old boy is brought to the emergency department because of a 3-day history of fever and severe wet cough. His temperature is 40.5°C (104.9°F), and respirations are 65/min. Crackles are heard over all lung fields. A photomicrograph of a silver-stained specimen obtained via bronchoalveolar lavage is shown. In addition to a lack of expression of human leukocyte antigen-DR molecules by lymphocytes, flow cytometry of a peripheral blood specimen will most likely show markedly decreased populations of which of the following cell types?
CorrectIncorrect -
Question 46 of 50
46. Question
A study is conducted to determine if concomitant oral administration of ciprofloxacin and sucralfate results in a decreased plasma ciprofloxacin concentration. Twelve subjects are enrolled in the study; six receive ciprofloxacin only, and six receive both treatments. The half-life is determined, and a 1-week washout period follows. Each subject then receives the other treatment, and half-life is measured again. Which of the following best describes the design of this study?
CorrectIncorrect -
Question 47 of 50
47. Question
A 28-year-old man is participating in a study of the effect of altitude and conditioning on combat readiness. He is a Special Forces sergeant in the army. While he exercises in an environmental chamber, the ambient Po2 is decreased from 160 mm Hg to 60 mm Hg. The decrease in ambient Po2 will most likely cause an increase in which of the following in this patient?
CorrectIncorrect -
Question 48 of 50
48. Question
A 52-year-old woman comes to the emergency department because of a 2-week history of progressive shortness of breath and fatigue. Her pulse is 102/min, respirations are 22/min, and blood pressure is 100/80 mm Hg. Physical examination shows muffled breath sounds. Echocardiography shows a large pericardial effusion. Pericardiocentesis yields cloudy, serosanguineous fluid. Analysis of the fluid shows an increased protein concentration, numerous RBCs, and a small number of WBCs, indicative of malignancy.
Metastasis from which of the following sites is the most likely cause of the findings in this patient?CorrectIncorrect -
Question 49 of 50
49. Question
A 29-year-old woman comes to the physician because of irregular menstrual periods since menarche at the age of 12 years. She is 160 cm (5 ft 3 in) tall and weighs 86 kg (190 lb); BMI is 34 kg/m2. She is evaluated, and the diagnosis of polycystic ovarian syndrome is made. After explaining the diagnosis, the physician discusses behavioral changes, including dietary modification and exercise as part of her treatment plan. Which of the following is most likely to result in patient adherence to this plan?
CorrectIncorrect -
Question 50 of 50
50. Question
A 72-year-old woman with coronary artery disease comes to the physician because of a 2-month history of progressive angina symptoms with exertion. Five months ago, the patient underwent stent placement for significant stenoses of the proximal anterior interventricular (left anterior descending) and right coronary arteries. She remained symptom-free for 3 months. Her pulse is 76/min and regular, and blood pressure is 135/85 mm Hg. An ECG at rest shows no abnormalities; an exercise stress test shows ST- segment changes in leads II, III, and aVF. Which of the following is the most likely cause of this patient’s recurrent angina symptoms?
CorrectIncorrect